100% satisfaction guarantee Immediately available after payment Both online and in PDF No strings attached
logo-home
MVU NURS 620 EXAM 4 2024/2025 WITH 100% ACCURATE SOLUTIONS $16.49   Add to cart

Exam (elaborations)

MVU NURS 620 EXAM 4 2024/2025 WITH 100% ACCURATE SOLUTIONS

 5 views  0 purchase
  • Course
  • MVU NURS 620
  • Institution
  • MVU NURS 620

MVU NURS 620 EXAM 4 2024/2025 WITH 100% ACCURATE SOLUTIONS

Preview 4 out of 33  pages

  • August 26, 2024
  • 33
  • 2024/2025
  • Exam (elaborations)
  • Questions & answers
  • MVU NURS 620
  • MVU NURS 620
avatar-seller
YANCHY
MVU NURS 620 EXAM 4 2024/2025 WITH
100% ACCURATE SOLUTIONS


Marsha, age 40, has been given a diagnosis of rheumatoid arthritis. She
asks you whether she should continue taking her birth control pills. You
tell her:
A. To check with her Rheumatologist
B. To Stop
C. To Continue
D. The dose will have to be altered - Precise Answer ✔✔C


Which of the following is a sexually transmitted infection?
A. Candida vaginitis
B. Trichomonal vaginitis
C. Atrophic vaginitis
D. Lactobacilli vaginitis - Precise Answer ✔✔B


Emergency contraception refers to:
A. An induced abortion in an emergency room (ER)
B. Quickly starting on birth control pills in anticipation
of sexual intercourse
C. Having a medroxyprogesterone (Depo Provera) in
the ER every 12 weeks

,D. Taking emergency contraceptive pills - Precise Answer ✔✔D


Sydney, age 21, is taking an oral contraceptive (OC). She complains of
acne. How should you adjust the estrogen in the OC?
A. Increase the estrogen content
B. Decrease the estrogen content
C. Delete the estrogen content
D. No adjustment should be made to the estrogen
content - Precise Answer ✔✔A


Joanne wants to use some form of birth control, but because she is
getting married next year, she wants to be able to stop the birth control
method after the wedding and have her fertility restored almost
immediately. Which method do you recommend for her?
A. Birth control
B. Vaginal Ring
C. Depot medroxyprogesterone acetate injections
D. Lea's shield - Precise Answer ✔✔D


First line treatment of Polycystic Ovary syndrome
A. A Bilateral Oophorectomy
B. Oral Testosterone therapy
C. A combination of diet modification, weight loss, and
stress management

,D. A laparoscopy with a bilateral wedge resection - Precise Answer
✔✔C


Jennifer, age 27, is complaining of lower abdominal pain. After doing
some laboratory studies, you find leukocytosis, an elevated erythrocyte
sedimentation rate, and an elevated C-reactive protein level. Which is
the most appropriate diagnosis
A. Ovarian cyst
B. Pelvic inflammatory disease
C. Tubal pregnancy
D. Diverticulitis - Precise Answer ✔✔B


Endometrial cancer, hirsutism, acne, breast cancer, increased risk of
diabetes, infertility, menstrual bleeding problems, and an increased risk
of cardiovascular disease are clinical consequences of:
A. Mastalgia
B. Menorrhagia
C. Endometriosis
D. Persistent anovulation - Precise Answer ✔✔D


A sexually active woman should be aware that genital herpes simplex
virus
A. May be transmitted to a partner or newborn even in the absence of
lesions because of viral shedding
B. Is suppressed during menstruation, physical or emotional stress,
immunosuppression, sexual intercourse, and pregnancy

, C. Included recurring outbreaks that last the same length of time as the
initial outbreak
D. Requires the use of condoms only during outbreaks - Precise Answer
✔✔A


A 27-year-old presents to your office for a Mirena (levonorgestrel
intrauterine system) insertion. She reports that her menses started 3 days
ago and is normal. How soon after the insertion will she be able to safely
rely on Mirena for contraception?
A. Immediately
B. After 48 hours
C. In 1 week
D. In 1 month - Precise Answer ✔✔A


A 26 year comes to your office to discuss birth control options. Her
history includes migraine headaches with aura while on combination
oral contraceptives in the past. She does not want to become pregnant.
Which of the following birth control options would be the best choice
for her?
A. Combined hormonal contraceptive pills
B. Ortho Evra Patch
C. Mirena Intrauterine device (IUD)
D. Vaginal Nuva Ring - Precise Answer ✔✔C


In a patient diagnosed with cervical gonococcal infection, you would
also suspect a co-infection with:

The benefits of buying summaries with Stuvia:

Guaranteed quality through customer reviews

Guaranteed quality through customer reviews

Stuvia customers have reviewed more than 700,000 summaries. This how you know that you are buying the best documents.

Quick and easy check-out

Quick and easy check-out

You can quickly pay through credit card or Stuvia-credit for the summaries. There is no membership needed.

Focus on what matters

Focus on what matters

Your fellow students write the study notes themselves, which is why the documents are always reliable and up-to-date. This ensures you quickly get to the core!

Frequently asked questions

What do I get when I buy this document?

You get a PDF, available immediately after your purchase. The purchased document is accessible anytime, anywhere and indefinitely through your profile.

Satisfaction guarantee: how does it work?

Our satisfaction guarantee ensures that you always find a study document that suits you well. You fill out a form, and our customer service team takes care of the rest.

Who am I buying these notes from?

Stuvia is a marketplace, so you are not buying this document from us, but from seller YANCHY. Stuvia facilitates payment to the seller.

Will I be stuck with a subscription?

No, you only buy these notes for $16.49. You're not tied to anything after your purchase.

Can Stuvia be trusted?

4.6 stars on Google & Trustpilot (+1000 reviews)

72841 documents were sold in the last 30 days

Founded in 2010, the go-to place to buy study notes for 14 years now

Start selling
$16.49
  • (0)
  Add to cart